Can exact square roots not be found?












9














I'm brushing up on some higher level maths for a programming project. I was going along and then I realized that I have absolutely no idea how square roots can be computed.



I mean sure, I've memorized a lot of perfect squares. But I wouldn't be able to get an exact answer from some arbitrary number like 432,434, would I?



I Googled how to calculate square roots and everything always points to it basically being based on algorithms which have a degree of error because they're more or less guesses.



I can't seem to find out why it's impossible to get an exact square root though. Like why can't you plug in $x$ to a function $f(x)$ and get the exact square root?



Very curious about this.










share|cite|improve this question




















  • 39




    What do you expect to be the exact "computed" value of $sqrt{2}$, say?
    – metamorphy
    Dec 20 at 10:38






  • 5




    Depending on how many bits you use for your floats, exact values do not exist for irrational numbers to computers. Increasing, for example, from 4 bit to 8 bit floats increases the accuracy, but does not eliminate rounding errors, even for rational numbers. For example $1+2^{-53}-1=2^{-53}$ obviously, but MATLAB for instance will tell you this sum is "exactly" zero
    – User123456789
    Dec 20 at 10:52








  • 32




    Forget about square roots; you can't even calculate 1/10 accurately using IEEE floats. If you are brushing up on your math for a project, you need to know the at least the bare minimum that all programmers must know to work with floating point numbers. If you have not read docs.oracle.com/cd/E19957-01/806-3568/ncg_goldberg.html do not do any more math programming until you have!
    – Eric Lippert
    Dec 20 at 14:43






  • 3




    @User123456789 exact binary values for computers don't exist even for many rational numbers. But one can represent many irrationals precisely if using another representation (instead of a single float).
    – Džuris
    Dec 20 at 14:50






  • 11




    Let's make things simpler. What do you expect to be the exact computed value of $frac{1}{9}$ ?
    – Brian
    Dec 20 at 15:25


















9














I'm brushing up on some higher level maths for a programming project. I was going along and then I realized that I have absolutely no idea how square roots can be computed.



I mean sure, I've memorized a lot of perfect squares. But I wouldn't be able to get an exact answer from some arbitrary number like 432,434, would I?



I Googled how to calculate square roots and everything always points to it basically being based on algorithms which have a degree of error because they're more or less guesses.



I can't seem to find out why it's impossible to get an exact square root though. Like why can't you plug in $x$ to a function $f(x)$ and get the exact square root?



Very curious about this.










share|cite|improve this question




















  • 39




    What do you expect to be the exact "computed" value of $sqrt{2}$, say?
    – metamorphy
    Dec 20 at 10:38






  • 5




    Depending on how many bits you use for your floats, exact values do not exist for irrational numbers to computers. Increasing, for example, from 4 bit to 8 bit floats increases the accuracy, but does not eliminate rounding errors, even for rational numbers. For example $1+2^{-53}-1=2^{-53}$ obviously, but MATLAB for instance will tell you this sum is "exactly" zero
    – User123456789
    Dec 20 at 10:52








  • 32




    Forget about square roots; you can't even calculate 1/10 accurately using IEEE floats. If you are brushing up on your math for a project, you need to know the at least the bare minimum that all programmers must know to work with floating point numbers. If you have not read docs.oracle.com/cd/E19957-01/806-3568/ncg_goldberg.html do not do any more math programming until you have!
    – Eric Lippert
    Dec 20 at 14:43






  • 3




    @User123456789 exact binary values for computers don't exist even for many rational numbers. But one can represent many irrationals precisely if using another representation (instead of a single float).
    – Džuris
    Dec 20 at 14:50






  • 11




    Let's make things simpler. What do you expect to be the exact computed value of $frac{1}{9}$ ?
    – Brian
    Dec 20 at 15:25
















9












9








9


2





I'm brushing up on some higher level maths for a programming project. I was going along and then I realized that I have absolutely no idea how square roots can be computed.



I mean sure, I've memorized a lot of perfect squares. But I wouldn't be able to get an exact answer from some arbitrary number like 432,434, would I?



I Googled how to calculate square roots and everything always points to it basically being based on algorithms which have a degree of error because they're more or less guesses.



I can't seem to find out why it's impossible to get an exact square root though. Like why can't you plug in $x$ to a function $f(x)$ and get the exact square root?



Very curious about this.










share|cite|improve this question















I'm brushing up on some higher level maths for a programming project. I was going along and then I realized that I have absolutely no idea how square roots can be computed.



I mean sure, I've memorized a lot of perfect squares. But I wouldn't be able to get an exact answer from some arbitrary number like 432,434, would I?



I Googled how to calculate square roots and everything always points to it basically being based on algorithms which have a degree of error because they're more or less guesses.



I can't seem to find out why it's impossible to get an exact square root though. Like why can't you plug in $x$ to a function $f(x)$ and get the exact square root?



Very curious about this.







calculus algebra-precalculus square-numbers programming






share|cite|improve this question















share|cite|improve this question













share|cite|improve this question




share|cite|improve this question








edited Dec 22 at 16:57









DaveInCaz

1798




1798










asked Dec 20 at 10:34









Edward Severinsen

18815




18815








  • 39




    What do you expect to be the exact "computed" value of $sqrt{2}$, say?
    – metamorphy
    Dec 20 at 10:38






  • 5




    Depending on how many bits you use for your floats, exact values do not exist for irrational numbers to computers. Increasing, for example, from 4 bit to 8 bit floats increases the accuracy, but does not eliminate rounding errors, even for rational numbers. For example $1+2^{-53}-1=2^{-53}$ obviously, but MATLAB for instance will tell you this sum is "exactly" zero
    – User123456789
    Dec 20 at 10:52








  • 32




    Forget about square roots; you can't even calculate 1/10 accurately using IEEE floats. If you are brushing up on your math for a project, you need to know the at least the bare minimum that all programmers must know to work with floating point numbers. If you have not read docs.oracle.com/cd/E19957-01/806-3568/ncg_goldberg.html do not do any more math programming until you have!
    – Eric Lippert
    Dec 20 at 14:43






  • 3




    @User123456789 exact binary values for computers don't exist even for many rational numbers. But one can represent many irrationals precisely if using another representation (instead of a single float).
    – Džuris
    Dec 20 at 14:50






  • 11




    Let's make things simpler. What do you expect to be the exact computed value of $frac{1}{9}$ ?
    – Brian
    Dec 20 at 15:25
















  • 39




    What do you expect to be the exact "computed" value of $sqrt{2}$, say?
    – metamorphy
    Dec 20 at 10:38






  • 5




    Depending on how many bits you use for your floats, exact values do not exist for irrational numbers to computers. Increasing, for example, from 4 bit to 8 bit floats increases the accuracy, but does not eliminate rounding errors, even for rational numbers. For example $1+2^{-53}-1=2^{-53}$ obviously, but MATLAB for instance will tell you this sum is "exactly" zero
    – User123456789
    Dec 20 at 10:52








  • 32




    Forget about square roots; you can't even calculate 1/10 accurately using IEEE floats. If you are brushing up on your math for a project, you need to know the at least the bare minimum that all programmers must know to work with floating point numbers. If you have not read docs.oracle.com/cd/E19957-01/806-3568/ncg_goldberg.html do not do any more math programming until you have!
    – Eric Lippert
    Dec 20 at 14:43






  • 3




    @User123456789 exact binary values for computers don't exist even for many rational numbers. But one can represent many irrationals precisely if using another representation (instead of a single float).
    – Džuris
    Dec 20 at 14:50






  • 11




    Let's make things simpler. What do you expect to be the exact computed value of $frac{1}{9}$ ?
    – Brian
    Dec 20 at 15:25










39




39




What do you expect to be the exact "computed" value of $sqrt{2}$, say?
– metamorphy
Dec 20 at 10:38




What do you expect to be the exact "computed" value of $sqrt{2}$, say?
– metamorphy
Dec 20 at 10:38




5




5




Depending on how many bits you use for your floats, exact values do not exist for irrational numbers to computers. Increasing, for example, from 4 bit to 8 bit floats increases the accuracy, but does not eliminate rounding errors, even for rational numbers. For example $1+2^{-53}-1=2^{-53}$ obviously, but MATLAB for instance will tell you this sum is "exactly" zero
– User123456789
Dec 20 at 10:52






Depending on how many bits you use for your floats, exact values do not exist for irrational numbers to computers. Increasing, for example, from 4 bit to 8 bit floats increases the accuracy, but does not eliminate rounding errors, even for rational numbers. For example $1+2^{-53}-1=2^{-53}$ obviously, but MATLAB for instance will tell you this sum is "exactly" zero
– User123456789
Dec 20 at 10:52






32




32




Forget about square roots; you can't even calculate 1/10 accurately using IEEE floats. If you are brushing up on your math for a project, you need to know the at least the bare minimum that all programmers must know to work with floating point numbers. If you have not read docs.oracle.com/cd/E19957-01/806-3568/ncg_goldberg.html do not do any more math programming until you have!
– Eric Lippert
Dec 20 at 14:43




Forget about square roots; you can't even calculate 1/10 accurately using IEEE floats. If you are brushing up on your math for a project, you need to know the at least the bare minimum that all programmers must know to work with floating point numbers. If you have not read docs.oracle.com/cd/E19957-01/806-3568/ncg_goldberg.html do not do any more math programming until you have!
– Eric Lippert
Dec 20 at 14:43




3




3




@User123456789 exact binary values for computers don't exist even for many rational numbers. But one can represent many irrationals precisely if using another representation (instead of a single float).
– Džuris
Dec 20 at 14:50




@User123456789 exact binary values for computers don't exist even for many rational numbers. But one can represent many irrationals precisely if using another representation (instead of a single float).
– Džuris
Dec 20 at 14:50




11




11




Let's make things simpler. What do you expect to be the exact computed value of $frac{1}{9}$ ?
– Brian
Dec 20 at 15:25






Let's make things simpler. What do you expect to be the exact computed value of $frac{1}{9}$ ?
– Brian
Dec 20 at 15:25












11 Answers
11






active

oldest

votes


















46














The square roots of non-perfect-square-integers are irrational numbers, which means that they have an infinite number of decimals, that do not repeat. So it would be a little tedious to write down the exact value...





Square roots can be computed, among others, by the so-called Heron's method (known BC), which is of the "guess" type, but converges extremely fast.






share|cite|improve this answer























  • However, numbers such as $sqrt{432434}$ can be written as a periodic continued fraction of this form: $$[a_0;a_1,a_2,dots,a_k,overline{a_{k+1},a_{k+2},dots,a_{k+m}}]$$
    – Jeppe Stig Nielsen
    Dec 21 at 17:30






  • 7




    @JeppeStigNielsen: it can also be written compactly as the positive solution of $z^2=432434$.
    – Yves Daoust
    Dec 21 at 18:29



















29














I understand that this question is actually about the representation of the square root in binary (float) or decimal notation.



However, that is not what these numbers are about themselves.



$sqrt 3$ is itself a perfectly valid number and an exact representation of the number that is the square root of 3. It can't be exactly represented as a binary or decimal number as these representations ar fairly limited (you can't even find an exact $1/3$ in either of those). But you can find it up to a certain precision.



Furthermore, these representations are actually used in computers. For example, Wolfram Mathematica will not replace $sqrt 3$ or $frac 2 {17}$ with an approximate representation which could screw up simplification later on. If you need the precision, you can exactly the same - store the numbers in a fancy structure that allows you to save the fact that it's a root of some number.






share|cite|improve this answer





























    9














    It depends what data types you have available in your programming environment. Typically, you’d be using double-precision floating-point numbers. But floating-point numbers are always rational numbers (quotient of two integers), and square roots are often not rational, so they can’t be represented exactly by floating-point values.



    If you have a more exotic set of data types, you may be able to represent some (but not all) square roots exactly.



    But there’s not really much point calculating square roots exactly unless our divisions and trig functions are also exact (which they are not, in floating-point math).






    share|cite|improve this answer





















    • Comments are not for extended discussion; this conversation has been moved to chat.
      – Aloizio Macedo
      Dec 22 at 14:54



















    6














    You can't find the exact value of square roots for non-square number. For the same reason you can't calculate the exact value of $pi$ or $e$.



    It's not a question of programming or approximation. Square roots of non-square numbers are irrational numbers.



    One of their properties is that you can't calculate an exact, numeric value for them, you can't represent them in decimal notation. You can represent them with a symbol or approximate them, but that's it.



    $sqrt 2$ is actually the most famous irrational number, and is the reason we discovered them.






    share|cite|improve this answer



















    • 10




      I wonder if, rather than saying you can't find the exact value, it would be more correct to say that you can't express the exact value in decimal notation (or similar notation in other number bases). If you allow other ways of representing values, e.g. a notation using the square root symbol, then the problem becomes trivial.
      – Michael Kay
      Dec 20 at 15:14










    • @MichaelKay It is still not trivial, because you need to show that there IS a "number" (however you choose to define of "number") that corresponds to the thing you are talking about. (Simple example: there is no REAL number which is the square root of $-1$).
      – alephzero
      Dec 20 at 15:28






    • 2




      How are you defining "square number" here? 6.25 isn't something I would think is usually regarded as a square number but it has a rational root
      – Martin Smith
      Dec 20 at 16:33










    • @MartinSmith Fair point. I don't know if I should defined more precisely to get just the ones that are irrational or say "most non-square numbers".
      – Teleporting Goat
      Dec 20 at 16:38



















    5














    Technically speaking, when you find $sqrt2 = 1.414 ...$, you're calculating the decimal expansion of the square root of 2. When people say things like "calculate the square root of 2", what they generally mean is to find this decimal expansion. That's because when people think of the term "number", what they generally think of is "number represented in decimal format". Since the decimal expansion has an infinite number of digits, it's impossible to calculate in its entirety. (Although, more technically speaking, decimal expansion is writing the number in terms of powers of ten, while floating point expresses numbers in terms of powers of two, but that distinction doesn't affect this particular issue).



    However, if we consider "number" as simply an abstract data class with particular methods, then it is possible to have a data class in which "square root of two" is a valid object, and an object whose square is exactly equal to two. In this context, the phrase "calculate the square root of two" would not have a clear meaning. We might have a print method that takes a number of digits, and outputs that number of digits. For instance, we might have that two.power(one.divide(two)).print(5) (i.e "print five digits after the decimal point of 2^(1/2)") gives the output 1.41421, but precise language would refer to this in terms of the method that is defined in the class, rather than just referring to it as "calculating" the square root of two. If you just type two.power(one.divide(two)), then depending on how the class is implemented, it may indeed "calculate" the floating point representation of $sqrt 2$, but it may represent the object in a more abstract form. In the former case, two.power(one.divide(two)).power(two) (that is, $(sqrt 2)^2)$) could be slightly less than 2, but in the latter case, it could very well return exactly 2. (It could also return exactly 2 in the former case, depending on how the rounding is set up).






    share|cite|improve this answer





























      2














      An irrational number cannot be equated to a finite series of rational numbers, only an infinite series of such.



      Assuming $x$ is rational, a fairly compact infinite series of rational numbers which equate to the square root and its reciprocal, that are derived from the generalized binomial theorem, are given below:



      $$sqrt{x} = sum_{k=0}^infty frac{1}{4^k}binom{2k}{k}left(1-frac{1}{x} right)^k$$
      and



      $$frac{1}{sqrt{x}} = -sum_{k=0}^infty frac{1}{4^k left( 2k-1right)}binom{2k}{k}left(1-frac{1}{x} right)^k$$



      where $x>1$. See here for proof.



      Unfortunately these series are much slower to converge to the desired values than other approximation/iterative methods available. Not even the most efficient algorithm will find you the exact value for a square root in your lifetime.






      share|cite|improve this answer































        2














        Square roots are typically computed by root finding; if you want to find $sqrt{x}$, you would use a root finding algorithm on



        $$g(t) = t^2 - x$$



        One such algorithm would be, for example, Newton's Method. Wikipedia has an entire page describing algorithms for root finding, and some more for computing square roots.



        The unifying theme behind these algorithms is that you define a sequence ${x_n}_{n in mathbb{N}}$ such that $x_n to x^*$, where $g(x^*) = 0$ and, depending on the algorithm and function, you can prove that it converges to the true root, and that it does so with a certain speed. The convergence properties usually depend on a good first approximation to where the root is, so not everything's rose colored. However, a priori this means that you can get arbitrarily close to the root and, for well behaved inputs, you will get the actual root (i.e. perfect squares).



        As many other answers described, computers work with a floating point representation, which are limited in what they can represent to a subset of the rational numbers. This means, in practice, that most numbers can't in fact be handled by your computer, and furthermore that you have to take several precautions when working with them; "What Every Computer Scientist Should Know About Floating-Point Arithmetic" is a pretty classic introduction, which is also friendly for math students.



        Computer Algebra Systems usually choose to instead work with a symbolic representation of math: instead of using actual numbers, implement the typical rules of math on symbols. In this approach, you would have a representation for $sqrt{2}$, and a rule that tells you $sqrt{x}^2 = x$; thus $sqrt{2}^2 = 2$. At the end, they do have to approximate, but they have the most accurate representation possible until the very last minute, which avoids many errors.



        If you are interested in reading more, "Numerical Analysis" by Richard L. Burden is a book I really liked when taking my Numerical Analysis class.






        share|cite|improve this answer





























          1














          Square roots(and all other roots and some other interesting functions) of some numbers are irrational numbers. It means that they have infinite trail of digits after a decimal point(or comma in some other languages) and these digits are not periodic. Your screen has a finite number of pixels so it cannot depict an infinite number of digits. Therefore you will have to omit some of them. Even if you depict 50000 first digits and omit the rest of infinity it's still not exactly precise.



          However don't think that such numbers are somehow unreal. They are used to measure physical quantities in given units. For instance if you have a square with a side exactly 1 meter long then its diagonal equals exactly square root of 2.






          share|cite|improve this answer










          New contributor




          Gherman is a new contributor to this site. Take care in asking for clarification, commenting, and answering.
          Check out our Code of Conduct.


















          • "It means that they have infinite trail of digits after a comma" or the decimal point if you're American. ;-)
            – Phil N DeBlanc
            Dec 20 at 12:27



















          0














          Depending on what you're trying to do with the square root once it's been computed, a pair of floating point numbers bracketing the true value might be good enough. This is called Interval arithmetic .



          Here's an example demonstrating the use of pyinterval (credit here for extracting the bits of a double in Python).



          from interval import interval, imath
          import struct

          # https://stackoverflow.com/a/23624284/931154
          def double_to_hex(f):
          return hex(struct.unpack('<Q', struct.pack('<d', f))[0])

          a = interval(10)
          b = imath.sqrt(a)

          print(b)
          print(double_to_hex(b[0][0]))
          print(double_to_hex(b[0][1]))


          When run, this script produces the following output, the two hexadecimal numbers are the bit patterns of the floats, so it's clear they aren't identical.



          interval([3.16227766017, 3.16227766017])
          0x40094c583ada5b52
          0x40094c583ada5b53


          Because isqrt (interval sqrt) has a dedicated implementation in the library, the interval produced is as narrow as possible, but functions do not produce narrow intervals in general. Successive operations tend to cause the interval to grow. It's also important to keep in mind that interval arithmetic is inherently pessimistic, especially if the same variable appears multiple times in the expression of interest.






          share|cite|improve this answer































            0














            Even supposing you could symbolically handle irrational numbers in your program, if x is a rational and f(x) can be irrational, there's no rigorously defined (i.e. finite) algorithm to implement f, whatever it may be. Square root is just one example of such f.



            You cannot express (using rational numbers) the square root of an arbitrary integer x with a mathematical tool less complicated than an infinite sum.



            That said, if you want a fancy, practical yet fascinating (that's opinionated) answer to your original problem, I'd suggest reading this: https://en.wikipedia.org/wiki/Fast_inverse_square_root






            share|cite|improve this answer





























              0














              You cannot always get exact square roots because of the nature of the numbers, for example square root of 2 is irrational meaning you have a infinity of decimals






              share|cite|improve this answer




















                protected by Aloizio Macedo Dec 22 at 14:54



                Thank you for your interest in this question.
                Because it has attracted low-quality or spam answers that had to be removed, posting an answer now requires 10 reputation on this site (the association bonus does not count).



                Would you like to answer one of these unanswered questions instead?














                11 Answers
                11






                active

                oldest

                votes








                11 Answers
                11






                active

                oldest

                votes









                active

                oldest

                votes






                active

                oldest

                votes









                46














                The square roots of non-perfect-square-integers are irrational numbers, which means that they have an infinite number of decimals, that do not repeat. So it would be a little tedious to write down the exact value...





                Square roots can be computed, among others, by the so-called Heron's method (known BC), which is of the "guess" type, but converges extremely fast.






                share|cite|improve this answer























                • However, numbers such as $sqrt{432434}$ can be written as a periodic continued fraction of this form: $$[a_0;a_1,a_2,dots,a_k,overline{a_{k+1},a_{k+2},dots,a_{k+m}}]$$
                  – Jeppe Stig Nielsen
                  Dec 21 at 17:30






                • 7




                  @JeppeStigNielsen: it can also be written compactly as the positive solution of $z^2=432434$.
                  – Yves Daoust
                  Dec 21 at 18:29
















                46














                The square roots of non-perfect-square-integers are irrational numbers, which means that they have an infinite number of decimals, that do not repeat. So it would be a little tedious to write down the exact value...





                Square roots can be computed, among others, by the so-called Heron's method (known BC), which is of the "guess" type, but converges extremely fast.






                share|cite|improve this answer























                • However, numbers such as $sqrt{432434}$ can be written as a periodic continued fraction of this form: $$[a_0;a_1,a_2,dots,a_k,overline{a_{k+1},a_{k+2},dots,a_{k+m}}]$$
                  – Jeppe Stig Nielsen
                  Dec 21 at 17:30






                • 7




                  @JeppeStigNielsen: it can also be written compactly as the positive solution of $z^2=432434$.
                  – Yves Daoust
                  Dec 21 at 18:29














                46












                46








                46






                The square roots of non-perfect-square-integers are irrational numbers, which means that they have an infinite number of decimals, that do not repeat. So it would be a little tedious to write down the exact value...





                Square roots can be computed, among others, by the so-called Heron's method (known BC), which is of the "guess" type, but converges extremely fast.






                share|cite|improve this answer














                The square roots of non-perfect-square-integers are irrational numbers, which means that they have an infinite number of decimals, that do not repeat. So it would be a little tedious to write down the exact value...





                Square roots can be computed, among others, by the so-called Heron's method (known BC), which is of the "guess" type, but converges extremely fast.







                share|cite|improve this answer














                share|cite|improve this answer



                share|cite|improve this answer








                edited Dec 21 at 14:40









                Community

                1




                1










                answered Dec 20 at 10:39









                Yves Daoust

                124k671221




                124k671221












                • However, numbers such as $sqrt{432434}$ can be written as a periodic continued fraction of this form: $$[a_0;a_1,a_2,dots,a_k,overline{a_{k+1},a_{k+2},dots,a_{k+m}}]$$
                  – Jeppe Stig Nielsen
                  Dec 21 at 17:30






                • 7




                  @JeppeStigNielsen: it can also be written compactly as the positive solution of $z^2=432434$.
                  – Yves Daoust
                  Dec 21 at 18:29


















                • However, numbers such as $sqrt{432434}$ can be written as a periodic continued fraction of this form: $$[a_0;a_1,a_2,dots,a_k,overline{a_{k+1},a_{k+2},dots,a_{k+m}}]$$
                  – Jeppe Stig Nielsen
                  Dec 21 at 17:30






                • 7




                  @JeppeStigNielsen: it can also be written compactly as the positive solution of $z^2=432434$.
                  – Yves Daoust
                  Dec 21 at 18:29
















                However, numbers such as $sqrt{432434}$ can be written as a periodic continued fraction of this form: $$[a_0;a_1,a_2,dots,a_k,overline{a_{k+1},a_{k+2},dots,a_{k+m}}]$$
                – Jeppe Stig Nielsen
                Dec 21 at 17:30




                However, numbers such as $sqrt{432434}$ can be written as a periodic continued fraction of this form: $$[a_0;a_1,a_2,dots,a_k,overline{a_{k+1},a_{k+2},dots,a_{k+m}}]$$
                – Jeppe Stig Nielsen
                Dec 21 at 17:30




                7




                7




                @JeppeStigNielsen: it can also be written compactly as the positive solution of $z^2=432434$.
                – Yves Daoust
                Dec 21 at 18:29




                @JeppeStigNielsen: it can also be written compactly as the positive solution of $z^2=432434$.
                – Yves Daoust
                Dec 21 at 18:29











                29














                I understand that this question is actually about the representation of the square root in binary (float) or decimal notation.



                However, that is not what these numbers are about themselves.



                $sqrt 3$ is itself a perfectly valid number and an exact representation of the number that is the square root of 3. It can't be exactly represented as a binary or decimal number as these representations ar fairly limited (you can't even find an exact $1/3$ in either of those). But you can find it up to a certain precision.



                Furthermore, these representations are actually used in computers. For example, Wolfram Mathematica will not replace $sqrt 3$ or $frac 2 {17}$ with an approximate representation which could screw up simplification later on. If you need the precision, you can exactly the same - store the numbers in a fancy structure that allows you to save the fact that it's a root of some number.






                share|cite|improve this answer


























                  29














                  I understand that this question is actually about the representation of the square root in binary (float) or decimal notation.



                  However, that is not what these numbers are about themselves.



                  $sqrt 3$ is itself a perfectly valid number and an exact representation of the number that is the square root of 3. It can't be exactly represented as a binary or decimal number as these representations ar fairly limited (you can't even find an exact $1/3$ in either of those). But you can find it up to a certain precision.



                  Furthermore, these representations are actually used in computers. For example, Wolfram Mathematica will not replace $sqrt 3$ or $frac 2 {17}$ with an approximate representation which could screw up simplification later on. If you need the precision, you can exactly the same - store the numbers in a fancy structure that allows you to save the fact that it's a root of some number.






                  share|cite|improve this answer
























                    29












                    29








                    29






                    I understand that this question is actually about the representation of the square root in binary (float) or decimal notation.



                    However, that is not what these numbers are about themselves.



                    $sqrt 3$ is itself a perfectly valid number and an exact representation of the number that is the square root of 3. It can't be exactly represented as a binary or decimal number as these representations ar fairly limited (you can't even find an exact $1/3$ in either of those). But you can find it up to a certain precision.



                    Furthermore, these representations are actually used in computers. For example, Wolfram Mathematica will not replace $sqrt 3$ or $frac 2 {17}$ with an approximate representation which could screw up simplification later on. If you need the precision, you can exactly the same - store the numbers in a fancy structure that allows you to save the fact that it's a root of some number.






                    share|cite|improve this answer












                    I understand that this question is actually about the representation of the square root in binary (float) or decimal notation.



                    However, that is not what these numbers are about themselves.



                    $sqrt 3$ is itself a perfectly valid number and an exact representation of the number that is the square root of 3. It can't be exactly represented as a binary or decimal number as these representations ar fairly limited (you can't even find an exact $1/3$ in either of those). But you can find it up to a certain precision.



                    Furthermore, these representations are actually used in computers. For example, Wolfram Mathematica will not replace $sqrt 3$ or $frac 2 {17}$ with an approximate representation which could screw up simplification later on. If you need the precision, you can exactly the same - store the numbers in a fancy structure that allows you to save the fact that it's a root of some number.







                    share|cite|improve this answer












                    share|cite|improve this answer



                    share|cite|improve this answer










                    answered Dec 20 at 15:08









                    Džuris

                    1,602926




                    1,602926























                        9














                        It depends what data types you have available in your programming environment. Typically, you’d be using double-precision floating-point numbers. But floating-point numbers are always rational numbers (quotient of two integers), and square roots are often not rational, so they can’t be represented exactly by floating-point values.



                        If you have a more exotic set of data types, you may be able to represent some (but not all) square roots exactly.



                        But there’s not really much point calculating square roots exactly unless our divisions and trig functions are also exact (which they are not, in floating-point math).






                        share|cite|improve this answer





















                        • Comments are not for extended discussion; this conversation has been moved to chat.
                          – Aloizio Macedo
                          Dec 22 at 14:54
















                        9














                        It depends what data types you have available in your programming environment. Typically, you’d be using double-precision floating-point numbers. But floating-point numbers are always rational numbers (quotient of two integers), and square roots are often not rational, so they can’t be represented exactly by floating-point values.



                        If you have a more exotic set of data types, you may be able to represent some (but not all) square roots exactly.



                        But there’s not really much point calculating square roots exactly unless our divisions and trig functions are also exact (which they are not, in floating-point math).






                        share|cite|improve this answer





















                        • Comments are not for extended discussion; this conversation has been moved to chat.
                          – Aloizio Macedo
                          Dec 22 at 14:54














                        9












                        9








                        9






                        It depends what data types you have available in your programming environment. Typically, you’d be using double-precision floating-point numbers. But floating-point numbers are always rational numbers (quotient of two integers), and square roots are often not rational, so they can’t be represented exactly by floating-point values.



                        If you have a more exotic set of data types, you may be able to represent some (but not all) square roots exactly.



                        But there’s not really much point calculating square roots exactly unless our divisions and trig functions are also exact (which they are not, in floating-point math).






                        share|cite|improve this answer












                        It depends what data types you have available in your programming environment. Typically, you’d be using double-precision floating-point numbers. But floating-point numbers are always rational numbers (quotient of two integers), and square roots are often not rational, so they can’t be represented exactly by floating-point values.



                        If you have a more exotic set of data types, you may be able to represent some (but not all) square roots exactly.



                        But there’s not really much point calculating square roots exactly unless our divisions and trig functions are also exact (which they are not, in floating-point math).







                        share|cite|improve this answer












                        share|cite|improve this answer



                        share|cite|improve this answer










                        answered Dec 20 at 10:45









                        bubba

                        30k32986




                        30k32986












                        • Comments are not for extended discussion; this conversation has been moved to chat.
                          – Aloizio Macedo
                          Dec 22 at 14:54


















                        • Comments are not for extended discussion; this conversation has been moved to chat.
                          – Aloizio Macedo
                          Dec 22 at 14:54
















                        Comments are not for extended discussion; this conversation has been moved to chat.
                        – Aloizio Macedo
                        Dec 22 at 14:54




                        Comments are not for extended discussion; this conversation has been moved to chat.
                        – Aloizio Macedo
                        Dec 22 at 14:54











                        6














                        You can't find the exact value of square roots for non-square number. For the same reason you can't calculate the exact value of $pi$ or $e$.



                        It's not a question of programming or approximation. Square roots of non-square numbers are irrational numbers.



                        One of their properties is that you can't calculate an exact, numeric value for them, you can't represent them in decimal notation. You can represent them with a symbol or approximate them, but that's it.



                        $sqrt 2$ is actually the most famous irrational number, and is the reason we discovered them.






                        share|cite|improve this answer



















                        • 10




                          I wonder if, rather than saying you can't find the exact value, it would be more correct to say that you can't express the exact value in decimal notation (or similar notation in other number bases). If you allow other ways of representing values, e.g. a notation using the square root symbol, then the problem becomes trivial.
                          – Michael Kay
                          Dec 20 at 15:14










                        • @MichaelKay It is still not trivial, because you need to show that there IS a "number" (however you choose to define of "number") that corresponds to the thing you are talking about. (Simple example: there is no REAL number which is the square root of $-1$).
                          – alephzero
                          Dec 20 at 15:28






                        • 2




                          How are you defining "square number" here? 6.25 isn't something I would think is usually regarded as a square number but it has a rational root
                          – Martin Smith
                          Dec 20 at 16:33










                        • @MartinSmith Fair point. I don't know if I should defined more precisely to get just the ones that are irrational or say "most non-square numbers".
                          – Teleporting Goat
                          Dec 20 at 16:38
















                        6














                        You can't find the exact value of square roots for non-square number. For the same reason you can't calculate the exact value of $pi$ or $e$.



                        It's not a question of programming or approximation. Square roots of non-square numbers are irrational numbers.



                        One of their properties is that you can't calculate an exact, numeric value for them, you can't represent them in decimal notation. You can represent them with a symbol or approximate them, but that's it.



                        $sqrt 2$ is actually the most famous irrational number, and is the reason we discovered them.






                        share|cite|improve this answer



















                        • 10




                          I wonder if, rather than saying you can't find the exact value, it would be more correct to say that you can't express the exact value in decimal notation (or similar notation in other number bases). If you allow other ways of representing values, e.g. a notation using the square root symbol, then the problem becomes trivial.
                          – Michael Kay
                          Dec 20 at 15:14










                        • @MichaelKay It is still not trivial, because you need to show that there IS a "number" (however you choose to define of "number") that corresponds to the thing you are talking about. (Simple example: there is no REAL number which is the square root of $-1$).
                          – alephzero
                          Dec 20 at 15:28






                        • 2




                          How are you defining "square number" here? 6.25 isn't something I would think is usually regarded as a square number but it has a rational root
                          – Martin Smith
                          Dec 20 at 16:33










                        • @MartinSmith Fair point. I don't know if I should defined more precisely to get just the ones that are irrational or say "most non-square numbers".
                          – Teleporting Goat
                          Dec 20 at 16:38














                        6












                        6








                        6






                        You can't find the exact value of square roots for non-square number. For the same reason you can't calculate the exact value of $pi$ or $e$.



                        It's not a question of programming or approximation. Square roots of non-square numbers are irrational numbers.



                        One of their properties is that you can't calculate an exact, numeric value for them, you can't represent them in decimal notation. You can represent them with a symbol or approximate them, but that's it.



                        $sqrt 2$ is actually the most famous irrational number, and is the reason we discovered them.






                        share|cite|improve this answer














                        You can't find the exact value of square roots for non-square number. For the same reason you can't calculate the exact value of $pi$ or $e$.



                        It's not a question of programming or approximation. Square roots of non-square numbers are irrational numbers.



                        One of their properties is that you can't calculate an exact, numeric value for them, you can't represent them in decimal notation. You can represent them with a symbol or approximate them, but that's it.



                        $sqrt 2$ is actually the most famous irrational number, and is the reason we discovered them.







                        share|cite|improve this answer














                        share|cite|improve this answer



                        share|cite|improve this answer








                        edited Dec 21 at 8:48

























                        answered Dec 20 at 13:39









                        Teleporting Goat

                        1686




                        1686








                        • 10




                          I wonder if, rather than saying you can't find the exact value, it would be more correct to say that you can't express the exact value in decimal notation (or similar notation in other number bases). If you allow other ways of representing values, e.g. a notation using the square root symbol, then the problem becomes trivial.
                          – Michael Kay
                          Dec 20 at 15:14










                        • @MichaelKay It is still not trivial, because you need to show that there IS a "number" (however you choose to define of "number") that corresponds to the thing you are talking about. (Simple example: there is no REAL number which is the square root of $-1$).
                          – alephzero
                          Dec 20 at 15:28






                        • 2




                          How are you defining "square number" here? 6.25 isn't something I would think is usually regarded as a square number but it has a rational root
                          – Martin Smith
                          Dec 20 at 16:33










                        • @MartinSmith Fair point. I don't know if I should defined more precisely to get just the ones that are irrational or say "most non-square numbers".
                          – Teleporting Goat
                          Dec 20 at 16:38














                        • 10




                          I wonder if, rather than saying you can't find the exact value, it would be more correct to say that you can't express the exact value in decimal notation (or similar notation in other number bases). If you allow other ways of representing values, e.g. a notation using the square root symbol, then the problem becomes trivial.
                          – Michael Kay
                          Dec 20 at 15:14










                        • @MichaelKay It is still not trivial, because you need to show that there IS a "number" (however you choose to define of "number") that corresponds to the thing you are talking about. (Simple example: there is no REAL number which is the square root of $-1$).
                          – alephzero
                          Dec 20 at 15:28






                        • 2




                          How are you defining "square number" here? 6.25 isn't something I would think is usually regarded as a square number but it has a rational root
                          – Martin Smith
                          Dec 20 at 16:33










                        • @MartinSmith Fair point. I don't know if I should defined more precisely to get just the ones that are irrational or say "most non-square numbers".
                          – Teleporting Goat
                          Dec 20 at 16:38








                        10




                        10




                        I wonder if, rather than saying you can't find the exact value, it would be more correct to say that you can't express the exact value in decimal notation (or similar notation in other number bases). If you allow other ways of representing values, e.g. a notation using the square root symbol, then the problem becomes trivial.
                        – Michael Kay
                        Dec 20 at 15:14




                        I wonder if, rather than saying you can't find the exact value, it would be more correct to say that you can't express the exact value in decimal notation (or similar notation in other number bases). If you allow other ways of representing values, e.g. a notation using the square root symbol, then the problem becomes trivial.
                        – Michael Kay
                        Dec 20 at 15:14












                        @MichaelKay It is still not trivial, because you need to show that there IS a "number" (however you choose to define of "number") that corresponds to the thing you are talking about. (Simple example: there is no REAL number which is the square root of $-1$).
                        – alephzero
                        Dec 20 at 15:28




                        @MichaelKay It is still not trivial, because you need to show that there IS a "number" (however you choose to define of "number") that corresponds to the thing you are talking about. (Simple example: there is no REAL number which is the square root of $-1$).
                        – alephzero
                        Dec 20 at 15:28




                        2




                        2




                        How are you defining "square number" here? 6.25 isn't something I would think is usually regarded as a square number but it has a rational root
                        – Martin Smith
                        Dec 20 at 16:33




                        How are you defining "square number" here? 6.25 isn't something I would think is usually regarded as a square number but it has a rational root
                        – Martin Smith
                        Dec 20 at 16:33












                        @MartinSmith Fair point. I don't know if I should defined more precisely to get just the ones that are irrational or say "most non-square numbers".
                        – Teleporting Goat
                        Dec 20 at 16:38




                        @MartinSmith Fair point. I don't know if I should defined more precisely to get just the ones that are irrational or say "most non-square numbers".
                        – Teleporting Goat
                        Dec 20 at 16:38











                        5














                        Technically speaking, when you find $sqrt2 = 1.414 ...$, you're calculating the decimal expansion of the square root of 2. When people say things like "calculate the square root of 2", what they generally mean is to find this decimal expansion. That's because when people think of the term "number", what they generally think of is "number represented in decimal format". Since the decimal expansion has an infinite number of digits, it's impossible to calculate in its entirety. (Although, more technically speaking, decimal expansion is writing the number in terms of powers of ten, while floating point expresses numbers in terms of powers of two, but that distinction doesn't affect this particular issue).



                        However, if we consider "number" as simply an abstract data class with particular methods, then it is possible to have a data class in which "square root of two" is a valid object, and an object whose square is exactly equal to two. In this context, the phrase "calculate the square root of two" would not have a clear meaning. We might have a print method that takes a number of digits, and outputs that number of digits. For instance, we might have that two.power(one.divide(two)).print(5) (i.e "print five digits after the decimal point of 2^(1/2)") gives the output 1.41421, but precise language would refer to this in terms of the method that is defined in the class, rather than just referring to it as "calculating" the square root of two. If you just type two.power(one.divide(two)), then depending on how the class is implemented, it may indeed "calculate" the floating point representation of $sqrt 2$, but it may represent the object in a more abstract form. In the former case, two.power(one.divide(two)).power(two) (that is, $(sqrt 2)^2)$) could be slightly less than 2, but in the latter case, it could very well return exactly 2. (It could also return exactly 2 in the former case, depending on how the rounding is set up).






                        share|cite|improve this answer


























                          5














                          Technically speaking, when you find $sqrt2 = 1.414 ...$, you're calculating the decimal expansion of the square root of 2. When people say things like "calculate the square root of 2", what they generally mean is to find this decimal expansion. That's because when people think of the term "number", what they generally think of is "number represented in decimal format". Since the decimal expansion has an infinite number of digits, it's impossible to calculate in its entirety. (Although, more technically speaking, decimal expansion is writing the number in terms of powers of ten, while floating point expresses numbers in terms of powers of two, but that distinction doesn't affect this particular issue).



                          However, if we consider "number" as simply an abstract data class with particular methods, then it is possible to have a data class in which "square root of two" is a valid object, and an object whose square is exactly equal to two. In this context, the phrase "calculate the square root of two" would not have a clear meaning. We might have a print method that takes a number of digits, and outputs that number of digits. For instance, we might have that two.power(one.divide(two)).print(5) (i.e "print five digits after the decimal point of 2^(1/2)") gives the output 1.41421, but precise language would refer to this in terms of the method that is defined in the class, rather than just referring to it as "calculating" the square root of two. If you just type two.power(one.divide(two)), then depending on how the class is implemented, it may indeed "calculate" the floating point representation of $sqrt 2$, but it may represent the object in a more abstract form. In the former case, two.power(one.divide(two)).power(two) (that is, $(sqrt 2)^2)$) could be slightly less than 2, but in the latter case, it could very well return exactly 2. (It could also return exactly 2 in the former case, depending on how the rounding is set up).






                          share|cite|improve this answer
























                            5












                            5








                            5






                            Technically speaking, when you find $sqrt2 = 1.414 ...$, you're calculating the decimal expansion of the square root of 2. When people say things like "calculate the square root of 2", what they generally mean is to find this decimal expansion. That's because when people think of the term "number", what they generally think of is "number represented in decimal format". Since the decimal expansion has an infinite number of digits, it's impossible to calculate in its entirety. (Although, more technically speaking, decimal expansion is writing the number in terms of powers of ten, while floating point expresses numbers in terms of powers of two, but that distinction doesn't affect this particular issue).



                            However, if we consider "number" as simply an abstract data class with particular methods, then it is possible to have a data class in which "square root of two" is a valid object, and an object whose square is exactly equal to two. In this context, the phrase "calculate the square root of two" would not have a clear meaning. We might have a print method that takes a number of digits, and outputs that number of digits. For instance, we might have that two.power(one.divide(two)).print(5) (i.e "print five digits after the decimal point of 2^(1/2)") gives the output 1.41421, but precise language would refer to this in terms of the method that is defined in the class, rather than just referring to it as "calculating" the square root of two. If you just type two.power(one.divide(two)), then depending on how the class is implemented, it may indeed "calculate" the floating point representation of $sqrt 2$, but it may represent the object in a more abstract form. In the former case, two.power(one.divide(two)).power(two) (that is, $(sqrt 2)^2)$) could be slightly less than 2, but in the latter case, it could very well return exactly 2. (It could also return exactly 2 in the former case, depending on how the rounding is set up).






                            share|cite|improve this answer












                            Technically speaking, when you find $sqrt2 = 1.414 ...$, you're calculating the decimal expansion of the square root of 2. When people say things like "calculate the square root of 2", what they generally mean is to find this decimal expansion. That's because when people think of the term "number", what they generally think of is "number represented in decimal format". Since the decimal expansion has an infinite number of digits, it's impossible to calculate in its entirety. (Although, more technically speaking, decimal expansion is writing the number in terms of powers of ten, while floating point expresses numbers in terms of powers of two, but that distinction doesn't affect this particular issue).



                            However, if we consider "number" as simply an abstract data class with particular methods, then it is possible to have a data class in which "square root of two" is a valid object, and an object whose square is exactly equal to two. In this context, the phrase "calculate the square root of two" would not have a clear meaning. We might have a print method that takes a number of digits, and outputs that number of digits. For instance, we might have that two.power(one.divide(two)).print(5) (i.e "print five digits after the decimal point of 2^(1/2)") gives the output 1.41421, but precise language would refer to this in terms of the method that is defined in the class, rather than just referring to it as "calculating" the square root of two. If you just type two.power(one.divide(two)), then depending on how the class is implemented, it may indeed "calculate" the floating point representation of $sqrt 2$, but it may represent the object in a more abstract form. In the former case, two.power(one.divide(two)).power(two) (that is, $(sqrt 2)^2)$) could be slightly less than 2, but in the latter case, it could very well return exactly 2. (It could also return exactly 2 in the former case, depending on how the rounding is set up).







                            share|cite|improve this answer












                            share|cite|improve this answer



                            share|cite|improve this answer










                            answered Dec 20 at 18:00









                            Acccumulation

                            6,7412617




                            6,7412617























                                2














                                An irrational number cannot be equated to a finite series of rational numbers, only an infinite series of such.



                                Assuming $x$ is rational, a fairly compact infinite series of rational numbers which equate to the square root and its reciprocal, that are derived from the generalized binomial theorem, are given below:



                                $$sqrt{x} = sum_{k=0}^infty frac{1}{4^k}binom{2k}{k}left(1-frac{1}{x} right)^k$$
                                and



                                $$frac{1}{sqrt{x}} = -sum_{k=0}^infty frac{1}{4^k left( 2k-1right)}binom{2k}{k}left(1-frac{1}{x} right)^k$$



                                where $x>1$. See here for proof.



                                Unfortunately these series are much slower to converge to the desired values than other approximation/iterative methods available. Not even the most efficient algorithm will find you the exact value for a square root in your lifetime.






                                share|cite|improve this answer




























                                  2














                                  An irrational number cannot be equated to a finite series of rational numbers, only an infinite series of such.



                                  Assuming $x$ is rational, a fairly compact infinite series of rational numbers which equate to the square root and its reciprocal, that are derived from the generalized binomial theorem, are given below:



                                  $$sqrt{x} = sum_{k=0}^infty frac{1}{4^k}binom{2k}{k}left(1-frac{1}{x} right)^k$$
                                  and



                                  $$frac{1}{sqrt{x}} = -sum_{k=0}^infty frac{1}{4^k left( 2k-1right)}binom{2k}{k}left(1-frac{1}{x} right)^k$$



                                  where $x>1$. See here for proof.



                                  Unfortunately these series are much slower to converge to the desired values than other approximation/iterative methods available. Not even the most efficient algorithm will find you the exact value for a square root in your lifetime.






                                  share|cite|improve this answer


























                                    2












                                    2








                                    2






                                    An irrational number cannot be equated to a finite series of rational numbers, only an infinite series of such.



                                    Assuming $x$ is rational, a fairly compact infinite series of rational numbers which equate to the square root and its reciprocal, that are derived from the generalized binomial theorem, are given below:



                                    $$sqrt{x} = sum_{k=0}^infty frac{1}{4^k}binom{2k}{k}left(1-frac{1}{x} right)^k$$
                                    and



                                    $$frac{1}{sqrt{x}} = -sum_{k=0}^infty frac{1}{4^k left( 2k-1right)}binom{2k}{k}left(1-frac{1}{x} right)^k$$



                                    where $x>1$. See here for proof.



                                    Unfortunately these series are much slower to converge to the desired values than other approximation/iterative methods available. Not even the most efficient algorithm will find you the exact value for a square root in your lifetime.






                                    share|cite|improve this answer














                                    An irrational number cannot be equated to a finite series of rational numbers, only an infinite series of such.



                                    Assuming $x$ is rational, a fairly compact infinite series of rational numbers which equate to the square root and its reciprocal, that are derived from the generalized binomial theorem, are given below:



                                    $$sqrt{x} = sum_{k=0}^infty frac{1}{4^k}binom{2k}{k}left(1-frac{1}{x} right)^k$$
                                    and



                                    $$frac{1}{sqrt{x}} = -sum_{k=0}^infty frac{1}{4^k left( 2k-1right)}binom{2k}{k}left(1-frac{1}{x} right)^k$$



                                    where $x>1$. See here for proof.



                                    Unfortunately these series are much slower to converge to the desired values than other approximation/iterative methods available. Not even the most efficient algorithm will find you the exact value for a square root in your lifetime.







                                    share|cite|improve this answer














                                    share|cite|improve this answer



                                    share|cite|improve this answer








                                    edited Dec 20 at 14:22

























                                    answered Dec 20 at 13:23









                                    James Arathoon

                                    1,346421




                                    1,346421























                                        2














                                        Square roots are typically computed by root finding; if you want to find $sqrt{x}$, you would use a root finding algorithm on



                                        $$g(t) = t^2 - x$$



                                        One such algorithm would be, for example, Newton's Method. Wikipedia has an entire page describing algorithms for root finding, and some more for computing square roots.



                                        The unifying theme behind these algorithms is that you define a sequence ${x_n}_{n in mathbb{N}}$ such that $x_n to x^*$, where $g(x^*) = 0$ and, depending on the algorithm and function, you can prove that it converges to the true root, and that it does so with a certain speed. The convergence properties usually depend on a good first approximation to where the root is, so not everything's rose colored. However, a priori this means that you can get arbitrarily close to the root and, for well behaved inputs, you will get the actual root (i.e. perfect squares).



                                        As many other answers described, computers work with a floating point representation, which are limited in what they can represent to a subset of the rational numbers. This means, in practice, that most numbers can't in fact be handled by your computer, and furthermore that you have to take several precautions when working with them; "What Every Computer Scientist Should Know About Floating-Point Arithmetic" is a pretty classic introduction, which is also friendly for math students.



                                        Computer Algebra Systems usually choose to instead work with a symbolic representation of math: instead of using actual numbers, implement the typical rules of math on symbols. In this approach, you would have a representation for $sqrt{2}$, and a rule that tells you $sqrt{x}^2 = x$; thus $sqrt{2}^2 = 2$. At the end, they do have to approximate, but they have the most accurate representation possible until the very last minute, which avoids many errors.



                                        If you are interested in reading more, "Numerical Analysis" by Richard L. Burden is a book I really liked when taking my Numerical Analysis class.






                                        share|cite|improve this answer


























                                          2














                                          Square roots are typically computed by root finding; if you want to find $sqrt{x}$, you would use a root finding algorithm on



                                          $$g(t) = t^2 - x$$



                                          One such algorithm would be, for example, Newton's Method. Wikipedia has an entire page describing algorithms for root finding, and some more for computing square roots.



                                          The unifying theme behind these algorithms is that you define a sequence ${x_n}_{n in mathbb{N}}$ such that $x_n to x^*$, where $g(x^*) = 0$ and, depending on the algorithm and function, you can prove that it converges to the true root, and that it does so with a certain speed. The convergence properties usually depend on a good first approximation to where the root is, so not everything's rose colored. However, a priori this means that you can get arbitrarily close to the root and, for well behaved inputs, you will get the actual root (i.e. perfect squares).



                                          As many other answers described, computers work with a floating point representation, which are limited in what they can represent to a subset of the rational numbers. This means, in practice, that most numbers can't in fact be handled by your computer, and furthermore that you have to take several precautions when working with them; "What Every Computer Scientist Should Know About Floating-Point Arithmetic" is a pretty classic introduction, which is also friendly for math students.



                                          Computer Algebra Systems usually choose to instead work with a symbolic representation of math: instead of using actual numbers, implement the typical rules of math on symbols. In this approach, you would have a representation for $sqrt{2}$, and a rule that tells you $sqrt{x}^2 = x$; thus $sqrt{2}^2 = 2$. At the end, they do have to approximate, but they have the most accurate representation possible until the very last minute, which avoids many errors.



                                          If you are interested in reading more, "Numerical Analysis" by Richard L. Burden is a book I really liked when taking my Numerical Analysis class.






                                          share|cite|improve this answer
























                                            2












                                            2








                                            2






                                            Square roots are typically computed by root finding; if you want to find $sqrt{x}$, you would use a root finding algorithm on



                                            $$g(t) = t^2 - x$$



                                            One such algorithm would be, for example, Newton's Method. Wikipedia has an entire page describing algorithms for root finding, and some more for computing square roots.



                                            The unifying theme behind these algorithms is that you define a sequence ${x_n}_{n in mathbb{N}}$ such that $x_n to x^*$, where $g(x^*) = 0$ and, depending on the algorithm and function, you can prove that it converges to the true root, and that it does so with a certain speed. The convergence properties usually depend on a good first approximation to where the root is, so not everything's rose colored. However, a priori this means that you can get arbitrarily close to the root and, for well behaved inputs, you will get the actual root (i.e. perfect squares).



                                            As many other answers described, computers work with a floating point representation, which are limited in what they can represent to a subset of the rational numbers. This means, in practice, that most numbers can't in fact be handled by your computer, and furthermore that you have to take several precautions when working with them; "What Every Computer Scientist Should Know About Floating-Point Arithmetic" is a pretty classic introduction, which is also friendly for math students.



                                            Computer Algebra Systems usually choose to instead work with a symbolic representation of math: instead of using actual numbers, implement the typical rules of math on symbols. In this approach, you would have a representation for $sqrt{2}$, and a rule that tells you $sqrt{x}^2 = x$; thus $sqrt{2}^2 = 2$. At the end, they do have to approximate, but they have the most accurate representation possible until the very last minute, which avoids many errors.



                                            If you are interested in reading more, "Numerical Analysis" by Richard L. Burden is a book I really liked when taking my Numerical Analysis class.






                                            share|cite|improve this answer












                                            Square roots are typically computed by root finding; if you want to find $sqrt{x}$, you would use a root finding algorithm on



                                            $$g(t) = t^2 - x$$



                                            One such algorithm would be, for example, Newton's Method. Wikipedia has an entire page describing algorithms for root finding, and some more for computing square roots.



                                            The unifying theme behind these algorithms is that you define a sequence ${x_n}_{n in mathbb{N}}$ such that $x_n to x^*$, where $g(x^*) = 0$ and, depending on the algorithm and function, you can prove that it converges to the true root, and that it does so with a certain speed. The convergence properties usually depend on a good first approximation to where the root is, so not everything's rose colored. However, a priori this means that you can get arbitrarily close to the root and, for well behaved inputs, you will get the actual root (i.e. perfect squares).



                                            As many other answers described, computers work with a floating point representation, which are limited in what they can represent to a subset of the rational numbers. This means, in practice, that most numbers can't in fact be handled by your computer, and furthermore that you have to take several precautions when working with them; "What Every Computer Scientist Should Know About Floating-Point Arithmetic" is a pretty classic introduction, which is also friendly for math students.



                                            Computer Algebra Systems usually choose to instead work with a symbolic representation of math: instead of using actual numbers, implement the typical rules of math on symbols. In this approach, you would have a representation for $sqrt{2}$, and a rule that tells you $sqrt{x}^2 = x$; thus $sqrt{2}^2 = 2$. At the end, they do have to approximate, but they have the most accurate representation possible until the very last minute, which avoids many errors.



                                            If you are interested in reading more, "Numerical Analysis" by Richard L. Burden is a book I really liked when taking my Numerical Analysis class.







                                            share|cite|improve this answer












                                            share|cite|improve this answer



                                            share|cite|improve this answer










                                            answered Dec 20 at 16:28









                                            Misguided

                                            685416




                                            685416























                                                1














                                                Square roots(and all other roots and some other interesting functions) of some numbers are irrational numbers. It means that they have infinite trail of digits after a decimal point(or comma in some other languages) and these digits are not periodic. Your screen has a finite number of pixels so it cannot depict an infinite number of digits. Therefore you will have to omit some of them. Even if you depict 50000 first digits and omit the rest of infinity it's still not exactly precise.



                                                However don't think that such numbers are somehow unreal. They are used to measure physical quantities in given units. For instance if you have a square with a side exactly 1 meter long then its diagonal equals exactly square root of 2.






                                                share|cite|improve this answer










                                                New contributor




                                                Gherman is a new contributor to this site. Take care in asking for clarification, commenting, and answering.
                                                Check out our Code of Conduct.


















                                                • "It means that they have infinite trail of digits after a comma" or the decimal point if you're American. ;-)
                                                  – Phil N DeBlanc
                                                  Dec 20 at 12:27
















                                                1














                                                Square roots(and all other roots and some other interesting functions) of some numbers are irrational numbers. It means that they have infinite trail of digits after a decimal point(or comma in some other languages) and these digits are not periodic. Your screen has a finite number of pixels so it cannot depict an infinite number of digits. Therefore you will have to omit some of them. Even if you depict 50000 first digits and omit the rest of infinity it's still not exactly precise.



                                                However don't think that such numbers are somehow unreal. They are used to measure physical quantities in given units. For instance if you have a square with a side exactly 1 meter long then its diagonal equals exactly square root of 2.






                                                share|cite|improve this answer










                                                New contributor




                                                Gherman is a new contributor to this site. Take care in asking for clarification, commenting, and answering.
                                                Check out our Code of Conduct.


















                                                • "It means that they have infinite trail of digits after a comma" or the decimal point if you're American. ;-)
                                                  – Phil N DeBlanc
                                                  Dec 20 at 12:27














                                                1












                                                1








                                                1






                                                Square roots(and all other roots and some other interesting functions) of some numbers are irrational numbers. It means that they have infinite trail of digits after a decimal point(or comma in some other languages) and these digits are not periodic. Your screen has a finite number of pixels so it cannot depict an infinite number of digits. Therefore you will have to omit some of them. Even if you depict 50000 first digits and omit the rest of infinity it's still not exactly precise.



                                                However don't think that such numbers are somehow unreal. They are used to measure physical quantities in given units. For instance if you have a square with a side exactly 1 meter long then its diagonal equals exactly square root of 2.






                                                share|cite|improve this answer










                                                New contributor




                                                Gherman is a new contributor to this site. Take care in asking for clarification, commenting, and answering.
                                                Check out our Code of Conduct.









                                                Square roots(and all other roots and some other interesting functions) of some numbers are irrational numbers. It means that they have infinite trail of digits after a decimal point(or comma in some other languages) and these digits are not periodic. Your screen has a finite number of pixels so it cannot depict an infinite number of digits. Therefore you will have to omit some of them. Even if you depict 50000 first digits and omit the rest of infinity it's still not exactly precise.



                                                However don't think that such numbers are somehow unreal. They are used to measure physical quantities in given units. For instance if you have a square with a side exactly 1 meter long then its diagonal equals exactly square root of 2.







                                                share|cite|improve this answer










                                                New contributor




                                                Gherman is a new contributor to this site. Take care in asking for clarification, commenting, and answering.
                                                Check out our Code of Conduct.









                                                share|cite|improve this answer



                                                share|cite|improve this answer








                                                edited Dec 20 at 13:56





















                                                New contributor




                                                Gherman is a new contributor to this site. Take care in asking for clarification, commenting, and answering.
                                                Check out our Code of Conduct.









                                                answered Dec 20 at 12:06









                                                Gherman

                                                1114




                                                1114




                                                New contributor




                                                Gherman is a new contributor to this site. Take care in asking for clarification, commenting, and answering.
                                                Check out our Code of Conduct.





                                                New contributor





                                                Gherman is a new contributor to this site. Take care in asking for clarification, commenting, and answering.
                                                Check out our Code of Conduct.






                                                Gherman is a new contributor to this site. Take care in asking for clarification, commenting, and answering.
                                                Check out our Code of Conduct.












                                                • "It means that they have infinite trail of digits after a comma" or the decimal point if you're American. ;-)
                                                  – Phil N DeBlanc
                                                  Dec 20 at 12:27


















                                                • "It means that they have infinite trail of digits after a comma" or the decimal point if you're American. ;-)
                                                  – Phil N DeBlanc
                                                  Dec 20 at 12:27
















                                                "It means that they have infinite trail of digits after a comma" or the decimal point if you're American. ;-)
                                                – Phil N DeBlanc
                                                Dec 20 at 12:27




                                                "It means that they have infinite trail of digits after a comma" or the decimal point if you're American. ;-)
                                                – Phil N DeBlanc
                                                Dec 20 at 12:27











                                                0














                                                Depending on what you're trying to do with the square root once it's been computed, a pair of floating point numbers bracketing the true value might be good enough. This is called Interval arithmetic .



                                                Here's an example demonstrating the use of pyinterval (credit here for extracting the bits of a double in Python).



                                                from interval import interval, imath
                                                import struct

                                                # https://stackoverflow.com/a/23624284/931154
                                                def double_to_hex(f):
                                                return hex(struct.unpack('<Q', struct.pack('<d', f))[0])

                                                a = interval(10)
                                                b = imath.sqrt(a)

                                                print(b)
                                                print(double_to_hex(b[0][0]))
                                                print(double_to_hex(b[0][1]))


                                                When run, this script produces the following output, the two hexadecimal numbers are the bit patterns of the floats, so it's clear they aren't identical.



                                                interval([3.16227766017, 3.16227766017])
                                                0x40094c583ada5b52
                                                0x40094c583ada5b53


                                                Because isqrt (interval sqrt) has a dedicated implementation in the library, the interval produced is as narrow as possible, but functions do not produce narrow intervals in general. Successive operations tend to cause the interval to grow. It's also important to keep in mind that interval arithmetic is inherently pessimistic, especially if the same variable appears multiple times in the expression of interest.






                                                share|cite|improve this answer




























                                                  0














                                                  Depending on what you're trying to do with the square root once it's been computed, a pair of floating point numbers bracketing the true value might be good enough. This is called Interval arithmetic .



                                                  Here's an example demonstrating the use of pyinterval (credit here for extracting the bits of a double in Python).



                                                  from interval import interval, imath
                                                  import struct

                                                  # https://stackoverflow.com/a/23624284/931154
                                                  def double_to_hex(f):
                                                  return hex(struct.unpack('<Q', struct.pack('<d', f))[0])

                                                  a = interval(10)
                                                  b = imath.sqrt(a)

                                                  print(b)
                                                  print(double_to_hex(b[0][0]))
                                                  print(double_to_hex(b[0][1]))


                                                  When run, this script produces the following output, the two hexadecimal numbers are the bit patterns of the floats, so it's clear they aren't identical.



                                                  interval([3.16227766017, 3.16227766017])
                                                  0x40094c583ada5b52
                                                  0x40094c583ada5b53


                                                  Because isqrt (interval sqrt) has a dedicated implementation in the library, the interval produced is as narrow as possible, but functions do not produce narrow intervals in general. Successive operations tend to cause the interval to grow. It's also important to keep in mind that interval arithmetic is inherently pessimistic, especially if the same variable appears multiple times in the expression of interest.






                                                  share|cite|improve this answer


























                                                    0












                                                    0








                                                    0






                                                    Depending on what you're trying to do with the square root once it's been computed, a pair of floating point numbers bracketing the true value might be good enough. This is called Interval arithmetic .



                                                    Here's an example demonstrating the use of pyinterval (credit here for extracting the bits of a double in Python).



                                                    from interval import interval, imath
                                                    import struct

                                                    # https://stackoverflow.com/a/23624284/931154
                                                    def double_to_hex(f):
                                                    return hex(struct.unpack('<Q', struct.pack('<d', f))[0])

                                                    a = interval(10)
                                                    b = imath.sqrt(a)

                                                    print(b)
                                                    print(double_to_hex(b[0][0]))
                                                    print(double_to_hex(b[0][1]))


                                                    When run, this script produces the following output, the two hexadecimal numbers are the bit patterns of the floats, so it's clear they aren't identical.



                                                    interval([3.16227766017, 3.16227766017])
                                                    0x40094c583ada5b52
                                                    0x40094c583ada5b53


                                                    Because isqrt (interval sqrt) has a dedicated implementation in the library, the interval produced is as narrow as possible, but functions do not produce narrow intervals in general. Successive operations tend to cause the interval to grow. It's also important to keep in mind that interval arithmetic is inherently pessimistic, especially if the same variable appears multiple times in the expression of interest.






                                                    share|cite|improve this answer














                                                    Depending on what you're trying to do with the square root once it's been computed, a pair of floating point numbers bracketing the true value might be good enough. This is called Interval arithmetic .



                                                    Here's an example demonstrating the use of pyinterval (credit here for extracting the bits of a double in Python).



                                                    from interval import interval, imath
                                                    import struct

                                                    # https://stackoverflow.com/a/23624284/931154
                                                    def double_to_hex(f):
                                                    return hex(struct.unpack('<Q', struct.pack('<d', f))[0])

                                                    a = interval(10)
                                                    b = imath.sqrt(a)

                                                    print(b)
                                                    print(double_to_hex(b[0][0]))
                                                    print(double_to_hex(b[0][1]))


                                                    When run, this script produces the following output, the two hexadecimal numbers are the bit patterns of the floats, so it's clear they aren't identical.



                                                    interval([3.16227766017, 3.16227766017])
                                                    0x40094c583ada5b52
                                                    0x40094c583ada5b53


                                                    Because isqrt (interval sqrt) has a dedicated implementation in the library, the interval produced is as narrow as possible, but functions do not produce narrow intervals in general. Successive operations tend to cause the interval to grow. It's also important to keep in mind that interval arithmetic is inherently pessimistic, especially if the same variable appears multiple times in the expression of interest.







                                                    share|cite|improve this answer














                                                    share|cite|improve this answer



                                                    share|cite|improve this answer








                                                    edited Dec 21 at 6:59

























                                                    answered Dec 21 at 6:53









                                                    Gregory Nisbet

                                                    493312




                                                    493312























                                                        0














                                                        Even supposing you could symbolically handle irrational numbers in your program, if x is a rational and f(x) can be irrational, there's no rigorously defined (i.e. finite) algorithm to implement f, whatever it may be. Square root is just one example of such f.



                                                        You cannot express (using rational numbers) the square root of an arbitrary integer x with a mathematical tool less complicated than an infinite sum.



                                                        That said, if you want a fancy, practical yet fascinating (that's opinionated) answer to your original problem, I'd suggest reading this: https://en.wikipedia.org/wiki/Fast_inverse_square_root






                                                        share|cite|improve this answer


























                                                          0














                                                          Even supposing you could symbolically handle irrational numbers in your program, if x is a rational and f(x) can be irrational, there's no rigorously defined (i.e. finite) algorithm to implement f, whatever it may be. Square root is just one example of such f.



                                                          You cannot express (using rational numbers) the square root of an arbitrary integer x with a mathematical tool less complicated than an infinite sum.



                                                          That said, if you want a fancy, practical yet fascinating (that's opinionated) answer to your original problem, I'd suggest reading this: https://en.wikipedia.org/wiki/Fast_inverse_square_root






                                                          share|cite|improve this answer
























                                                            0












                                                            0








                                                            0






                                                            Even supposing you could symbolically handle irrational numbers in your program, if x is a rational and f(x) can be irrational, there's no rigorously defined (i.e. finite) algorithm to implement f, whatever it may be. Square root is just one example of such f.



                                                            You cannot express (using rational numbers) the square root of an arbitrary integer x with a mathematical tool less complicated than an infinite sum.



                                                            That said, if you want a fancy, practical yet fascinating (that's opinionated) answer to your original problem, I'd suggest reading this: https://en.wikipedia.org/wiki/Fast_inverse_square_root






                                                            share|cite|improve this answer












                                                            Even supposing you could symbolically handle irrational numbers in your program, if x is a rational and f(x) can be irrational, there's no rigorously defined (i.e. finite) algorithm to implement f, whatever it may be. Square root is just one example of such f.



                                                            You cannot express (using rational numbers) the square root of an arbitrary integer x with a mathematical tool less complicated than an infinite sum.



                                                            That said, if you want a fancy, practical yet fascinating (that's opinionated) answer to your original problem, I'd suggest reading this: https://en.wikipedia.org/wiki/Fast_inverse_square_root







                                                            share|cite|improve this answer












                                                            share|cite|improve this answer



                                                            share|cite|improve this answer










                                                            answered Dec 21 at 15:02









                                                            Kafein

                                                            611




                                                            611























                                                                0














                                                                You cannot always get exact square roots because of the nature of the numbers, for example square root of 2 is irrational meaning you have a infinity of decimals






                                                                share|cite|improve this answer


























                                                                  0














                                                                  You cannot always get exact square roots because of the nature of the numbers, for example square root of 2 is irrational meaning you have a infinity of decimals






                                                                  share|cite|improve this answer
























                                                                    0












                                                                    0








                                                                    0






                                                                    You cannot always get exact square roots because of the nature of the numbers, for example square root of 2 is irrational meaning you have a infinity of decimals






                                                                    share|cite|improve this answer












                                                                    You cannot always get exact square roots because of the nature of the numbers, for example square root of 2 is irrational meaning you have a infinity of decimals







                                                                    share|cite|improve this answer












                                                                    share|cite|improve this answer



                                                                    share|cite|improve this answer










                                                                    answered Dec 22 at 11:58









                                                                    oren revenge

                                                                    329112




                                                                    329112

















                                                                        protected by Aloizio Macedo Dec 22 at 14:54



                                                                        Thank you for your interest in this question.
                                                                        Because it has attracted low-quality or spam answers that had to be removed, posting an answer now requires 10 reputation on this site (the association bonus does not count).



                                                                        Would you like to answer one of these unanswered questions instead?



                                                                        Popular posts from this blog

                                                                        Список кардиналов, возведённых папой римским Каликстом III

                                                                        Deduzione

                                                                        Mysql.sock missing - “Can't connect to local MySQL server through socket”